Daily Drills 67 - Section 67 - Question 1

The conclusion of the columnist's argument can be properly drawn if which one of the following is assumed?

nimakian801 November 10, 2019

Can you explain this?

Confused why this isn't must be true

Replies
Create a free account to read and take part in forum discussions.

Already have an account? log in

Irina November 10, 2019

@nimakian,

This question stem indicates a strengthen with sufficient assumption question. A sufficient assumption allows us to draw a conclusion/ guarantees the truth of the conclusion, whereas in a must be true question we are required to make an inference/ reach a conclusion assuming all the premises are true.

Other question stems that indicate a sufficient assumption question include:

"The conclusion above follows logically if which one of the following is assumed?”
“Which of the following, if true, enables the conclusion to be properly drawn?”
“The conclusion above is properly drawn if which of the following is assumed?”

nimakian801 November 10, 2019

Oh I see. So it's a different between the statements in the passage making something true versus something in the answer choice if true helps us conclude the above. Did I get this right?

Irina November 11, 2019

@nimakian801,

You got it!

Starr-Latimer February 5, 2023

Why wouldn't this be a strengthen with necessary question? I thought the "must be assumed" implied a necessary factor.

Emil-Kunkin February 5, 2023

I don't see any language about must be assumed, which would indeed make this a necessary assumption. This is asking us what would make it so the argument can be valid, which is a sufficient assumption.